Which order pair makes both inequalities true? Y<-2x+3Y< x-2

Answers

Answer 1

The ordered pair which makes both inequalities true are (3, 0).

Inequalities are mathematical expressions where neither side is equal.

Contrary to equations, we compare two values in inequality.

Less than, greater than, or not equal to signs are used in place of the equal to sign in between.

Let us consider the ordered pair (3, 0)

In the first inequality

y > -2x + 3

Substituting the values of x and y

0 > -2(3) + 3

0 > -6 + 3

0 > -3

In the second inequality

y < x - 2

Substituting the value of x and y

0 < 3 - 2

0 < 1

Therefore, the ordered pair which makes both inequalities true are (3, 0).

To learn more about inequalities visit:

https://brainly.com/question/20383699

#SPJ9


Related Questions

2. The total sales for June at Jim's Candy Store were $7,785. The total sales for June and Julywere $12,603, what were the total sales for July?ExplorerPlanSolve:Examine:Answer:

Answers

[tex]\begin{gathered} \text{June sales = 7785, } \\ \text{ June sales + July sales = 12603 } \\ \text{July sales = 12603 - june sales} \\ \text{July sales = 12603 - 7785} \\ \text{July sales = }4818 \end{gathered}[/tex]

A dairy produced 8.1 liters of milk in 2 hours. How much milk, on average, did the dairyproduce per hour?

Answers

To answer this question, we need to find the unit rate in this case. For this, we need to divide the given liters by the hours. Then, we have:

[tex]\frac{8.1l}{2h}=4.05\frac{l}{h}[/tex]

Then, the dairy produces 4.05 liters per hour.

can you answer 3 please show a table graph and work

Answers

To solve the question, choose values for x to find its corresponding image y. Then, plot the points and connect them.

Step 01: Choosing x = -2.

Substituting x by -2 in the equation:

[tex]\begin{gathered} y=2\cdot3^{-2} \\ y=2\cdot(\frac{1}{3})^2 \\ y=2\cdot\frac{1}{9} \\ y=\frac{2}{9} \end{gathered}[/tex]

So, the first point is (-2, 2/9).

Step 02: Choosing x = -1.

Substituting x by -1 in the equation:

[tex]\begin{gathered} y=2\cdot3^{-1} \\ y=2\cdot(\frac{1}{3})^1 \\ y=2\cdot\frac{1}{3} \\ y=\frac{2}{3} \end{gathered}[/tex]

So, the second point is (-1, 2/3).

Step 03: Choosing x = 0.

Substituting x by 0 in the equation:

[tex]\begin{gathered} y=2\cdot3^0 \\ y=2\cdot1 \\ y=2 \end{gathered}[/tex]

So, the third point is (0, 2).

Step 04: Choosing x = 1.

Substituting x by 1 in the equation:

[tex]\begin{gathered} y=2\cdot3^1 \\ y=2\cdot3 \\ y=6 \end{gathered}[/tex]

So, the fourth point is (1, 6).

Step 05: Write the points in a table.

x y (x, y)

-2 2/9 (-2, 2/9)

-1 2/3 (-1, 2/3)

0 2 (0, 2)

1 6 (1, 6)

Step 06: Plot the points and connect them.

The figure below shows the points and the graph.

Done! Your question is solved!

4^3/(-12+ 2^2)

(2x2)^2 + (-5 x 2 x 3 ) + 2
[tex] \frac{4^3/(-12 +2^2)}{(2x3)^2 +(-5 x2 x 3)+2} [/tex]
i need help!!​

Answers

Answer:

-1

Step-by-step explanation:

[tex]\frac{\frac{64}{-12+4}}{(6)^2+(-30)+2} \\ \\ =\frac{\frac{64}{-8}}{36-30+2} \\ \\ =\frac{-8}{8} \\ \\ =-1[/tex]

Don Stone obtained an $8.500 installment loan at 14% for 42 months. The loan's balance after 26 payments is 3.733.55. What is the interest for payment 27?

Answers

Given:

The unpaid balance after the 26 payments is $3,733.55.

Therefore, the interest for payment 27 will be

[tex]14\text{ \% of \$3733.55}[/tex]

Evaluating

[tex]\frac{14}{100}\times3733.55=0.14\times3733.55=522.697\approx522.70(nearest\text{ cent)}[/tex]

Hence, the interest for payment 27 is $522.70.

The two lines y y = x and y = x + 1 are parallel lines.
True
False

Answers

By definition, the Slope-Intercept Form of the equation of a line is:

[tex]y=mx+b[/tex]

Where "m" is the slope of the line and "b" is the y-intercept.

The equation of a line that passes through the Origin has the following form:

[tex]y=mx[/tex]

Where "m" is the slope of the line.

In this case, you have the first line that passes through the Origin:

[tex]y=x[/tex]

You can identify that its slope is:

[tex]m_1=1[/tex]

You also know the second equation, which is written in Slope-Intercept form:

[tex]y=x+1[/tex]

You can identify that:

[tex]\begin{gathered} m_2=1 \\ b=1 \end{gathered}[/tex]

By definition, the slopes of parallel lines are equal. Then, since:

[tex]m_1=m_2[/tex]

These lines are parallel.

The answer is: True.

Given a triangle ABC at points A = ( - 6, 3 ) B = ( - 4, 7 ) C = ( - 2, 3 ), and a first transformation of up 2 and right 3, and a second transformation of down 1 and left 6, what would be the location of the final point B'' ?

Answers

Answer:

B'' = (-7, 8)

Explanation:

The points of the triangle are:

A = ( - 6, 3 ) B = ( - 4, 7 ) C = ( - 2, 3 )

The first transformation:

2 units up and 3 units right

B' = (-4+3, 7+2)

B' = (-1, 9)

Second transformation:

1 unit down, 6 units left

B'' = (-1-6, 9-1)

B'' = (-7, 8)

a sea turtle can swim at rate of 20 miles per hour. How many feet per hour can a sea turtle swim

Answers

The rate at which turtle can swim is 20 miles per hour or 20 miles in one hour.

For conversion, 1 mile is equal to 5280 foot.

Convert 20 miles per hour in foot per hour.

[tex]\begin{gathered} 20\text{ mile per hour=20 miles per hour}\times\frac{5280\text{ foot per hour}}{1\text{ mile per hour}} \\ =20\cdot5280\text{ foot per hour} \\ =105600\text{ foot per hour} \end{gathered}[/tex]

So answer is 105600 foot per hour.

Question 3 of 102 PointsWhat is the midpoint of the segment shown below?(-1,2)(73)O A. (6,3)O B. (3,3)O C. (3.)O D. (6,5)10

Answers

[tex]undefined[/tex]

The engine of a car has a displacement of 460 cubic inches. What is the displacement in cubic feet? Round your answer to 2 places.

Answers

Explanation

To find the displacement in cubic feet, divide the volume value by 1728.

[tex]\frac{460}{1728}=0.27[/tex]

Answer: 0.27 cubic feet

Rain equation of a hyperbola given the foci and the asymptotes

Answers

Explanation

The equation for a hyperbola that opens up and down has the following general form:

[tex]\frac{(y-k)^2}{a^2}-\frac{(x-h)^2}{b^2}=1[/tex]

Where the foci of the hyperbola are located at (h,k+c) and (h,k-c) with c given by:

[tex]c^2=a^2+b^2[/tex]

And asymptotes with slopes given by a/b and -a/b.

The hyperbola with the equation that we have to find has these two foci:

[tex](3,2-\sqrt{26})\text{ and }(3,2+\sqrt{26})[/tex]

This means that:

[tex]\begin{gathered} (h,k-c)=(3,2-\sqrt{26}) \\ (h,k+c)=(3,2+\sqrt{26}) \end{gathered}[/tex]

So we get h=3, k=2 and c=√26.

The slope of the asymptotes have to be 5 and -5 which means that:

[tex]\frac{a}{b}=5[/tex]

Using the value of c we have:

[tex]c^2=26=a^2+b^2[/tex]

So we have two equation for a and b. We can take the first one and multiply b to both sides:

[tex]\begin{gathered} \frac{a}{b}\cdot b=5b \\ a=5b \end{gathered}[/tex]

And we use this in the second equation:

[tex]\begin{gathered} 26=(5b)^2+b^2=25b^2+b^2 \\ 26=26b^2 \end{gathered}[/tex]

We divide both sides by 26:

[tex]\begin{gathered} \frac{26}{26}=\frac{26b^2}{26} \\ b^2=1 \end{gathered}[/tex]

Which implies that b=1. Then a is equal to:

[tex]a=5b=5\cdot1=5[/tex]Answer

Now that we have found a, b, h and k we can write the equation of the hyperbola. Then the answer is:

[tex]\frac{(y-2)^2}{5^2}-\frac{(x-3)^2}{1^2}=1[/tex]

Write 12.5% as a decimal.

Answers

12.5% as a decimal is 0.125.

To convert a percentage in to a decimal, we divide the percentage by 100:

[tex]12.5\div100=0.125[/tex]

A student bought a calculator and a textbook for a course in algebra. He told his friend that the total cost was $165 (without tax) and thatthe calculator cost $25 more than thrice the cost of the textbook. Whatwas the cost of each item? Let x = the cost of a calculator andy = the cost of the textbook. The corresponding modeling system is { x = 3y + 25x + y =Solve the system by using the method of= 165substitution

Answers

We know that the calculator price (x) was 25 more than 3 times the price of the textbook (y).

This can be represented as:

[tex]x=3y+25[/tex]

We also know that the sum of the prices of the two items is equal to $165:

[tex]x+y=165[/tex]

We have to solve this system of equations with the method of substitution.

We can use the first equation, as we have already clear the value of x, to substitute x in the second equation and then solve for y:

[tex]\begin{gathered} x+y=165 \\ (3y+25)+y=165 \\ 4y+25=165 \\ 4y=165-25 \\ 4y=140 \\ y=\frac{140}{4} \\ y=35 \end{gathered}[/tex]

With the value of y we can calculate x using the first equation:

[tex]\begin{gathered} x=3y+25 \\ x=3\cdot35+25 \\ x=105+25 \\ x=130 \end{gathered}[/tex]

Answer: the solution as ordered pair is (x,y) = (130, 35)

the amount of money a worker earns is directly proportional to the number of hours worked.at this rate.thag worker makes $104 in an 8 hour shift.weite a direct variation equation that relates the earnings (e) of the worker to the number of hours (n) worked. use the correct lowercase variables given ,and use no spaces

Answers

the amount of money a worker earns is directly proportional to the number of hours worked.at this rate.thag worker makes $104 in an 8 hour shift.weite a direct variation equation that relates the earnings (e) of the worker to the number of hours (n) worked. use the correct lowercase variables given ,and use no spaces​

we know that

A relationship between two variables, x, and y, represent a proportional variation if it can be expressed in the form e=kn

In this problem we have

e -----> amount of money a worker earns

n -----> number of hours worked

k is the constant of proportionality

k=e/n

Find the value of k

we have

For n=8 hours, e=$104

sibstitute

k=104/8

k=$13 per hour

The linear equation is

e=13n

Section 1- Question 1Ryan is solving the equation - 6x = 12 by completing the square. What number should be added to both sides of the equation to complete the square?

Answers

Solution:

Given the equation below

[tex]x^2-6x=12[/tex]

Applying the completing the square method

Where the general form of a quadratic equation is

[tex]ax^2+bx+c=0[/tex]

For the completing square method,

[tex]Add\text{ }(\frac{b}{2})^2\text{ to both sides of the equation}[/tex]

Where

[tex]b=-6[/tex]

The number that should be added to both sides of the equation to complete the square is

[tex]=(\frac{-6}{2})^2=(-3)^2=9[/tex]

Hence, the number is 9 (option B)

Simplify the expression. 2m - 8 - 2m - 1

Answers

[tex]\begin{gathered} 2m-8-2m-1 \\ 2m-2m-8-1 \\ -8-1 \\ -9 \end{gathered}[/tex]

I NEED HELP WITH THIS ASAP 100 POINTS IF SOMEONE GETS THIS RIGHT.
Question 12(Multiple Choice Worth 2 points)
(Interior and Exterior Angles MC)

For triangle XYZ, m∠X = (4g + 13)° and the exterior angle to ∠X measures (3g + 48)°. Find the measure of ∠X and its exterior angle.

Interior angle = 48°; exterior angle = 74.25°
Interior angle = 74.25°; exterior angle = 48°
Interior angle = 81°; exterior angle = 99°
Interior angle = 99°; exterior angle = 81°

Answers

Answer:

Interior angle = 81°; exterior angle = 99°.

Step-by-step explanation:

For triangle XYZ:

m∠X = (4g + 13)° exterior angle to ∠X = (3g + 48)°

Angle X and its exterior angle form a straight line.

Angles on a straight line sum to 180°.

Therefore:

⇒ (4g + 13)° + (3g + 48)° = 180°

⇒ 4g + 13 + 3g + 48 = 180

⇒ 7g + 61 = 180

⇒ 7g + 61 - 61 = 180 - 61

⇒ 7g = 119

⇒ 7g ÷ 7 = 119 ÷ 7

⇒ g = 17

To find the measure of ∠X and its exterior angle, substitute the found value of g into the angle expressions:

⇒ m∠X = (4(17) + 13)°

⇒ m∠X = (68 + 13)°

⇒ m∠X = 81°

⇒ exterior angle to ∠X = (3(17) + 48)°

⇒ exterior angle to ∠X = (51 + 48)°

⇒ exterior angle to ∠X = 99°

Therefore:

Interior angle = 81°Exterior angle = 99°

Answer: C

Step-by-step explanation: did the practice test!

If Ellen's gross pay for a two-week period is $1680.00, what is her net pay?O $1606.92O $168.00O $1341.48O $1478.40

Answers

It's important to know that the gross pay refers to money before taxes, while the net pay refers to money after deductions.

Hence, the net payment must be less than $1,680.

Hence, the answer is $1,606.92.

I need help with 4 problems

Answers

1)

[tex]c^2=5^2+5^2[/tex]

then the solution is

[tex]c=\sqrt[]{25+25}=\sqrt[]{50}=5\sqrt[]{2}\approx7.1[/tex]

Complete each equation in order to obtain the indicated solution

Answers

Question 13.

Part (a).

Given the solution:

All real numbers

We have the expression:

3(4x + 2) = ________

Let's complete the equation in order to obtain the indicated solution.

For a solution to be all real numbers, the equation must be true.

hence, we have:

[tex]\begin{gathered} 3(4x+2)=3(4x+2) \\ \end{gathered}[/tex]

After solving we have:

0 = 0

This means the system has infinitely many solutions, therefore, the solution is all real numbers.

ANSWER:

3(4x + 2) = 3(4x + 2)

Determine if the following statement is true or false regarding sets A and B.A = {3, 5, 7, 9, 11, 13}B = {3, 5, 11, 13}Every element of A is also an element of B.Choose the correct answer below.FalseTrue

Answers

We have the following sets:

A = {3, 5, 7, 9, 11, 13}

B = {3, 5, 11, 13}

If we look closely, all the elements of B are in A. But each element of A does not belong to B, therefore the statement is totally false.

Question 1 Business Analytics

Answers

The responses to the linear optimization questions are;

Question 1

The optimal daily profit is $380

Question 2

The combination of x and y that yield the optimal value is the option;

x = 0, y = 3

What is linear optimization or optimization?

Linear programming is a method by which the optimal solution can be obtained from a model represented mathematically and in which the constraints of the model have linear relationships.

Question 1

Let B represent the number of bear claws, C the almond-filled croissant, F represent the flour, Y represent the amount of yeast and A represent the number of almonds.

The amount of ingredient per each produce is therefore;

B = 6·F + 1·Y + 2·A

C = 3·F + 1·Y + 4A

The amount of ingredient available for the days production is as follows;

Ingredient available; 6600·F + 1400·Y + 4800·A

The constraints are therefore

6·B + 3·C ≤ 6,600

B + C ≤ 1,400

2·B + 4·C ≤ 4800

The maximizing function is therefore;

Profit = 0.2·B + 0.3·F

The equations of the lines are therefore;

B = 1,100 - 0.5·C

B = 1400 - C

B = 2400 - 2·C

The vertices of the feasible region are;

(0, 1100), (600, 800), (1000, 400), 1200, 0)

The values of the maximizing function at the vertices of the feasible region are;

Profit, P = 0.2×1100 + 3×0 = 220

At point (600, 800), P = 0.2×800 + 0.3×600 = 340

At point (1000, 400), P = 0.2×400 + 0.3×1000 = 380

At point (1200, 0), P = 0.2×0 + 0.3×1200 = 360

The maximum profit is $380, obtained when 400 Bear claws and 1000 almond filled croissants are produced

Question 2

Maximize $3·x + $15·y

Subject to the following constraints;

2·x + 4·y ≤ 12

5·x + 2·y ≤ 10

x, y ≥ 10

The equations are therefore;

4·y ≤ 12 - 2·x

y ≤ 3 - 0.5·x...(1)

5·x + 2·y ≤ 10

2·y ≤ 10 - 5·x

y ≤ 5 - 2.5·x...(2)

x ≥ 10, y ≥ 10

The coordinates of the vertices of the feasible region are;

(0, 3), (1, 2.5), and (2, 0)

The values of the maximizing function are therefore;

At (0, 3), M = $3 × 0 + $15 × 3 = $45

At (1, 2.5), M = $3 × 1 + $15 × 2.5 = $40.5

At (2, 0), M = $3 × 2 + $15 × 0 = $6

The combination of x and y that yield the optimum is therefore;

(x, y) = (0, 3)

x = 0, and y = 3

Learn more about linear optimization or programming here:

https://brainly.com/question/28315344

#SPJ1

write (2 to the power of -1) to the power of 3 with the same base but one exponent

Answers

[tex]\frac{1}{8}[/tex]

Explanation

Step 1

[tex](2^{-1})^3[/tex]

remember

[tex]\begin{gathered} a^{-n}=\frac{1}{a^n} \\ (a^n)^m=a^{n\cdot m} \\ (\frac{a}{b})^m=\frac{a^m}{b^m} \end{gathered}[/tex]

Step 2

solve

[tex]\begin{gathered} (2^{-1})^3 \\ (2^{-1})^3=(\frac{1}{2^1})^3=(\frac{1^3}{2^3})=\frac{1}{8} \end{gathered}[/tex]

Using the data in the image could you help with this question State some possible causes of the error in your measured value. What techniques could be used to correct it?

Answers

Answer:

Step-by-step explanation:

A box contains four red marbles three green marbles and to blue marbles one marble is removed from the box and it's not replaced another marble is drawn from the box does the following represent in and a independent event

Answers

The correct option is Yes, which is option A

Why?

The reason is that the probability that marble is removed from the box does not affect the probability that a marble is drawn from the same box. i.e the two events do not affect each other

natural number is also a whole number.TrueFalse

Answers

Answer

The statement istrue.

Natural numbers are also whole numbers.

Explanation

Natural numbers are counting numbers.

They are the numbers that are numerically used to count things.

Hence, all natural numbers (counting numbers) are whole numbers.

Hope this Helps!!!

Differentiatey = -8 In x

Answers

Given:

[tex]y=-8lnx[/tex]

Let's differentiate the equation.

To differentiate since -8 is constant with resppect to x, the derivative will be:

[tex]\begin{gathered} \frac{d}{dx}(-8lnx) \\ \\ =-8\frac{d}{dx}(ln(x)) \end{gathered}[/tex]

Where:

derivative of ln(x) with respect to x = 1/x

Thus, we have:

[tex]\begin{gathered} -8\frac{1}{x} \\ \\ =-\frac{8}{x} \end{gathered}[/tex]

ANSWER:

[tex]-\frac{8}{x}[/tex]

Rewrite each equation in slope intercept form, if necessary. Then determine whether the lines are parallel. Explain3x+4y = 86x+3y = 6Are these lines parallel?A.B.C.D.(look at image for answer choices)

Answers

We can rewrite the next equations in the slope-intercept form:

The first equation:

[tex]3x+4y=8\Rightarrow4y=8-3x\Rightarrow y=\frac{8}{4}-\frac{3}{4}x\Rightarrow y=2-\frac{3}{4}x\Rightarrow y=-\frac{3}{4}x+2[/tex]

The second equation:

[tex]6x+3y=6\Rightarrow3y=6-6x\Rightarrow y=\frac{6}{3}-\frac{6}{3}x\Rightarrow y=2-2x\Rightarrow y=-2x+2_{}[/tex]

As we can see, the slope of the first line is m = -3/4, and the slope of the second line is m = -2. Then, since the slope is different, these lines are not parallel (Option C).

7:20 A.M to 9:49 A.M

Answers

We can add the minutes from 7:20 AM to the next hour (8:00 AM), that is 40 minutes.

Then, from 8:00 AM to 9:00 AM we have 60 more minutes.

Then, from 9:00 AM to 9:49 AM we have 49 additional minutes.

We add all three segments as:

[tex]40+60+49=149[/tex]

As 60 minutes is 1 hour, 120 minutes is 2 hours. Then, 149 minutes are 2 hours and 29 minutes.

Answer: the time elapsed us 149 minutes (or 2 hours and 29 minutes)

A line is graphed on the coordinate plane below.Line y = -2 +2 will be graphed on the same coordinate plane to create a system of equations.What is the solution to that system of equations?4A (-2,4)B (0-4)C (2,-4)0 (4,-2)Rod End TeeFlagOptionsBackNext

Answers

Solution:

Step 1: Find the equation of the line in the graph.

Two points the line pass through are (0, -4) and (2, -3)

Thus,

[tex]\begin{gathered} x_1=0,y_1=-4 \\ x_2=2,y_2=-3 \end{gathered}[/tex][tex]\begin{gathered} The\text{ equation of the line can be calculated with the formula} \\ \frac{y_2-y_1}{x_2-x_1}=\frac{y-y_1}{x-x_1} \\ \\ \frac{-3-(-4)}{2-0}=\frac{y-(-4)}{x-0} \\ \\ \frac{-3+4}{2}=\frac{y+4}{x} \\ \frac{1}{2}=\frac{y+4}{x} \end{gathered}[/tex][tex]\begin{gathered} 2(y+4)=x \\ 2y+8=x \\ 2y=x-8 \end{gathered}[/tex]

The equation of the graph is 2y = x - 8

Step 2:

Solve the two equations simultaneously to detemine the solution to the systems of equations

2y = x - 8 ------------------------equation (1)

y = -x + 2 ----------------------equation (2)

Add both equations to eliminate x

2y + y = x - 8 + (-x) + 2

3y = x -8-x+2

3y = -8 + 2

3y = -6

y = -6/3

y = -2

Substitute y = -2 into equation (2)

y = -x + 2

-2 = -x + 2

-2 -2 = -x

-4 = -x

-x = -4

Divide both sides by -1

x = 4

Hence, the solution to the system of equations is (4, -2)

The correct option is option D

Other Questions
A box of cereal costs $3.59 for 30 oz. Find the unit cost, round to the nearest hundredth. ? A meteorologist collected data about wind speed in a city, in miles per hour, on consecutive days of a month. Her data is shown using the dot plot. Create a box plot to represent the data. (1 point)dot plot titled Monthly Wind Speed and number line from 9 to 10 in increments of 1 tenth labeled Wind Speed (in miles per hour) with zero dots over 9, 1 dot over 9 and 1 tenth, 2 dots over 9 and 2 tenths, 1 dot over 9 and 3 tenths, 3 dots over 9 and 4 tenths, zero dots over 9 and 5 tenths, 1 dot over 9 and 6 tenths, 2 dots over 9 and 7 tenths, 1 dot over 9 and 8 tenths, zero dots over 9 and 9 tenths, and zero dots over box plot with minimum value 9 and 2 tenths, lower quartile 9 and 3 tenths, median 9 and 5 tenths, upper quartile 9 and 8 tenths, and maximum value 9 and 9 tenthsbox plot with minimum value 9 and 1 tenth, lower quartile 9 and 2 tenths, median 9 and 4 tenths, upper quartile 9 and 7 tenths, and maximum value 9 and 8 tenthsbox plot with minimum value 9 and 1 tenth, lower quartile 9 and 3 tenths, median 9 and 4 tenths, upper quartile 9 and 6 tenths, and maximum value 9 and 8 tenthsbox plot with minimum value 9 and 1 tenth, lower quartile 9 and 2 tenths, median 9 and 5 tenths, upper quartile 9 and 7 tenths, and maximum value 9 and 8 tenths 3(p-5)+2sp=12s=3solve. Pls help: find the rational expression state any restrictions on the variable Solve the given equation over the interval [0, 2.2): 2 cos2 x + cos x + 15 = 0.X = 0 and x = 2.0T57x= - and x=66There are no real valued solutions for the equation.T371x= and x =2 Dennis invest $4000 into an account that pays at a 3.5% interest rate compounded continuously. How many years will it take until Dennis has $6000 in his account? Round your answer to the nearest year In a city 20% of the cars are electric, 16% of the cars are red, and 14% of the cars are red electriccars. If a car is randomly picked and found to be red, what is the probability that this car is electric?Enter your answer as a decimal number rounded to TWO digits after the decimal point, like 0.12.DO NOT enter it like 12% or 12. (b) the central bank of econland buys $50,000 of government bonds from canary bank. what is the dollar value of the change in the monetary base? explain using numbers. find the length of one side of a solid cube given that itsvolume is 8 cm cubetotal surface area is 54 cm square. Find the slope of the line. Write your answer in simplest form.-5-4-3-2-1(1, -3)(-5,-4)The slope is45y1 x 5. Given: Line segment AD bisects I need help to do these composition of functions. I have a photo if needed.g(a)=a-1f(a)=3a-1Find (gf)(1) If you solved the following system by substitution, which of these could be yourcombined equation?y = 3x - 44x + 3y = 1 Complete each equation below so that it shows equivalent fractions. help meeeeeeeeeeeeeee pleaseeeeeee Please help me It would mean all lot to me :( An organ pipe that is closed at one end has a length of 3 m. What is the second longest harmonic wavelength for sound waves in this pipe A periodic wave has a frequency of 3.2 Hz. What is the wave period?answer in:____ s 4. Find the equationof a line with a slopeof 3 and through(2,9) oracion con ingenteplsssssssssssss